Đến nội dung

maitienluat nội dung

Có 183 mục bởi maitienluat (Tìm giới hạn từ 21-04-2020)



Sắp theo                Sắp xếp  

#504414 $\cos [(k-1)x], \cos (kx) \in \mathbb{Q}$

Đã gửi bởi maitienluat on 06-06-2014 - 11:25 trong Số học

Ta có nhận xét rằng nếu $\cos x$ là số hữu tỉ thì $\cos (kx)$ cũng là số hữu tỉ, với $k$ là số nguyên dương (do $\cos (kx)=P_k(\cos x)$, với $P_k(x)$ là đa thức Chebyshev loại I với bậc k, là một đa thức hệ số nguyên).

Vì vậy yêu cầu của đề bài tương đương với việc chứng minh hệ đồng dư sau có nghiệm

$\left\{\begin{matrix} n \equiv 0 &(mod k) \\ n \equiv 1 &(mod (k-1)) \end{matrix}\right.$

Nhưng hệ này luôn có nghiệm theo định lý thặng dư trung hoa ( chẳng hạn lấy $n=k^2$ ). Ta có đpcm.




#494953 $\sum_{i=0}^{2n} (-1)^i \frac{\b...

Đã gửi bởi maitienluat on 24-04-2014 - 20:32 trong Bất đẳng thức - Cực trị

Cho $n$ là số nguyên dương và $x$ là số thực dương. Chứng minh rằng:

$$\frac {\binom{2n}{0}}{x}-\frac {\binom{2n}{1}}{x+1}+\frac {\binom{2n}{2}}{x+2}-...+ \frac {\binom{2n}{2n}}{x+2n} > 0$$

với $\binom{n}{k}=\frac {n!}{k!(n-k)!}$




#492272 Trận 7 - Số học

Đã gửi bởi maitienluat on 11-04-2014 - 21:25 trong Thi giải toán Marathon Chuyên toán 2014

Bổ đề: Cho $x$ là số nguyên dương. Khi đó $x^2+1$ không có ước nguyên tố $p$ có dạng $4k+3$.

C/m: Giả sử tồn tại số nguyên tố $p$ như vậy. Khi đó $x^2 \equiv -1 (mod p)$

Theo định lý Fermat nhỏ : $(-1)^{2k+1} \equiv (x^2)^{2k+1} \equiv x^{p-1} \equiv 1 (mod p) $

Suy ra $ -1 \equiv 1 (mod p)$ hay $p=2$ (vô lí). Bổ đề được chứng minh.

Trở lại bài toán. 

Do bài toán không đổi khi thay $x$ bởi $-x$ nên ta có thể giả sử $x \geq 0$. Tương tự $y \geq 0$.

Ta chứng minh trong 2 số $x,y$ có ít nhất một số chẵn.

Thật vậy. Giả sử $x,y$ đều lẻ. Khi đó ta có $z^2+1 = x^2y^2-x^2-y^2 \equiv -1 \equiv 3 (mod 4)$

Mà $z^2+1 \equiv 1,2 (mod 4)$ nên vô lí. Vậy trong 2 số $x,y$ có ít nhất 1 số chẵn, giả sử là $x$.

Ta có $z^2+1 = x^2y^2-x^2-y^2+1 = (x^2-1)(y^2-1) $

Do $x$ chẵn nên $x^2-1 \equiv 3 (mod 4) $ nên nếu $x^2-1 > 2$ hay $x>2$ thì $x^2-1$ có có ước nguyên tố $p$ dạng $4k+3$. Nhưng khi đó $p$ là ước của $z^2+1$, mâu thuẫn theo bổ đề.

Suy ra $x \leq 1$.

Nếu $x=1$ thì $z^2+1 = 0$ , vô nghiệm.

Nếu $x=0$ thì $y^2+z^2 = 0 \Rightarrow y=z=0$.

Thử lại thấy $(x,y,z)=(0,0,0)$ thỏa. Vậy pt có nghiệm duy nhất $x=y=z=0$




#489290 Trận 6 - Phương trình và hệ phương trình

Đã gửi bởi maitienluat on 28-03-2014 - 21:13 trong Thi giải toán Marathon Chuyên toán 2014

Bài làm:

 

Hệ đã cho tương đương với $\left\{\begin{matrix} |x-1|.|y+1|=1 & \\ |x-1|-m.|y+1| \leq m-1 & \end{matrix}\right.$

Đặt $a=|x-1|,b=|y-1|$ thì $a \geq 0, b \geq 0$ và hệ trở thành $\left\{\begin{matrix} ab=1 & (1)\\ a-mb \leq m-1 & (2) \end{matrix}\right.$

Từ $(1)$ ta suy ra $a>0$. Do đó $(2) \Leftrightarrow a^2-mab \leq (m-1)a$

$\Leftrightarrow a^2-(m-1)a-m \leq 0$ (do $ab=1$)

$\Leftrightarrow (a+1)(a-m) \leq 0$

$\Rightarrow a \leq m$ (do $a+1 >1>0$)

Biện luận: Nếu $m \leq 0$ thì hệ bất phương trình đã cho vô nghiệm .

                 Nếu $m >0$ thì hệ đã cho có nghiệm (và có vô số nghiệm).




#485200 Trận 4 - Đa thức, phương trình hàm

Đã gửi bởi maitienluat on 28-02-2014 - 21:37 trong Thi giải toán Marathon Chuyên toán 2014

Bài làm:

 

Giả sử tồn tại hàm $f$ thoả mãn yêu cầu bài toán.

Trong  $(*)$ cho $x=y=0$ ta được $2f(0)=(f(0))^2 \Rightarrow f(0)=0$ hoặc $f(0)=2$.

  $\bullet$ TH1:  $f(0) = 0$

Trong $(*)$ cho $x=y$ được $f(xy)=f(x).f(y)$ (1)

Do đó từ $(*)$ suy ra $f(x-y) = f(x) - f(y) , \forall x,y \in \mathbb{R} $ (2)

Cho $x=0$ trong (2) thu được $f(-y) = -f(y)$.

Từ đó $(2) \Leftrightarrow f(x-y) = f(x) + f(-y)$ hay $f(x+y) = f(x) + f(y)$ (3).

Đặt $f(1)=a$. Trong $(3)$ cho $y=1$ được $f(y+1)=f(y)+a$.

Từ đây quy nạp ta được $f(n)=an , \forall n \in \mathbb{N}$.

Trong $(2)$ cho $x,y \in \mathbb{N}$ ta được $axy = a^2.xy$. Suy ra $a=0$ hoặc $a=1$

Nếu $a=0$ thì trong $(2)$ cho $y=1, x \in \mathbb{R}$ được $f(x)=0 \forall x \in \mathbb{R}$

Nếu $a=1$ thì ta thu được $f(n)=n , \forall n \in \mathbb{N}$

Lại có $f(-x)=-f(x)$ nên $f(n)=n , \forall n \in \mathbb{Z}$

Bây giờ với số hữu tỉ $x= \frac {p} {q}, p,q \in \mathbb{Z} , q \neq 0 $ bất kì, trong $(2)$ cho $x=\frac {p} {q},y=q$ thu được $p= f( \frac {p} {q} .q) = f(\frac {p} {q}) .f(q) = q.f(\frac {p} {q}) \Rightarrow f(\frac {p} {q}) = \frac {p} {q}$

Vậy $f(x)=x , \forall x \in \mathbb{Q}$.

Ta chứng minh rằng $f$ tăng.

Trong (2) lấy $x=y$ suy ra $f(x^2)=(f(x))^2$ , suy ra $f(x) \geq 0 , \forall x \geq 0$.

Với 2 số thực $x<y$ bất kì, từ $(3)$ suy ra $f(y)-f(x) = f(y-x) \geq 0$, vậy $f(x)$ tăng.

Với  mỗi số thực $x$ bất kì, ta chọn 2 dãy số hữu tỉ $(u_n) , (v_n)$ sao cho $u_n \leq x \leq v_n , \forall n$ và $\lim u_n = \lim v_n = x$. Do $f(x)$ tăng nên $u_n = f(u_n) \leq f(x) \leq f(v_n) = v_n , \forall n$.

Cho $n\rightarrow +\infty$ thu được $x \leq f(x) \leq x \Rightarrow f(x) = x$.

  $\bullet$ TH2: $f(0)=2$

Trong $(*)$ cho $y=0$ thu được $f(x) = 2 \forall x$.

Tóm lại ta thu được $f(x) = 0 , f(x) = 2, f(x) =x$.

Thử lại thấy cả $3$ hàm trên đều thoả.

Kết luận: $f(x)=0 , f(x) =2, f(x) =x$




#481909 Trận 3 - Tổ hợp rời rạc

Đã gửi bởi maitienluat on 08-02-2014 - 12:15 trong Thi giải toán Marathon Chuyên toán 2014

Mở rộng 2: Tổng quát cho số thành phố là $n \geq 3$ bất kì

Ta chỉ cần giải quyết cho trường hợp $n=3k+1$ và $n=3k+2$

Tương tự như bài làm và mở rộng 1, ta có số đường đi không vượt quá $P=a+b+ab+c(a+b)$ với $a+b+c=n-1$

Và $P \leq -3d^2 + 2dn$ với $d=\frac {a+b} {2}$

Xét $n=3k+1$.

Ta có $-3d^2+2dn=-3(d- \frac {n} {3})^2 +\frac {n^2} {3} \leq \frac {n^2} {3} = \frac {9k^2+6k+1} {3} = 3k^2+2k+\frac {1} {3}$

Do $a,b,c$ nguyên nên $P$ nguyên, suy ra $P \leq 3k^2+2k$

Ta chỉ ra một cách xây dựng đường đi.

Chia $3k+1$ thành phố thành 4 nhóm $A,B,C,D$ với các nhóm $A,B,C$ đều có $k$ thành phố và nhóm $D$ có 1 thành phố $M$, Với bộ 3 thành phố $X,Y,Z$ trong đó $X \in A, Y \in B, Z \in C$ ta xây dựng đường đi theo quy tắc $X \Rightarrow Y \Rightarrow Z \Rightarrow X$. Sau đó ta xây dựng đường đi từ mọi thành phố thuộc nhóm $A$ tới thành phố $M$ và từ thành phố $M$ tới mọi thành phố thuộc nhóm $C$.Dễ thấy cách xây dựng này thoả mãn và có đúng $3k^2+2k$ đường đi.

Xét $n=3k+2$.

Tương tự như trường hợp $n=3k+1$, ta có $-3d^2+2dn \leq 3k^2+4k+ \frac {4} {3}$, nên $P \leq 3k^2+4k+1$.

Ta chỉ ra một cách xây dựng.

Chia $3k+2$ thành phố thành 4 nhóm $A,B,C,D$ với các nhóm $A,B,C$ đều có $k$ thành phố và nhóm $D$ có 2 thành phố $M$ và $N$. Với bộ 3 thành phố $X,Y,Z$ trong đó $X \in A, Y \in B, Z \in C$ ta xây dựng đường đi theo quy tắc $X \Rightarrow Y \Rightarrow Z \Rightarrow X$. Sau đó ta xây dựng đường đi từ mọi thành phố thuộc nhóm $A$ tới thành phố $M$ và từ thành phố $M$ tới mọi thành phố thuộc nhóm $C$. Ta tiếp tục xây dựng đường đi từ mọi thành phố thuộc nhóm $C$ đến $N$, từ $N$ đến mọi thành phố thuộc nhóm $B$ và từ $N$ đến $M$. Dễ kiểm tra cách xây này thoả và có đúng $3k^2+4k+1$ đường đi.

Tóm lại, trong mọi trường hợp, số đường đi lớn nhất có thể là $\left \lfloor \frac{n^2}{3} \right \rfloor$ , ở đó $n$ là số thành phố.

 

 




#481901 Trận 3 - Tổ hợp rời rạc

Đã gửi bởi maitienluat on 08-02-2014 - 11:36 trong Thi giải toán Marathon Chuyên toán 2014

Mở rộng 1: Ta sẽ thay số thành phố $210$ bới $n=3k$ bất kì.

Với lập luận tương tự như trong bài làm, ta có số con đường một chiều có thể xây dựng không vượt quá $P=a+b+ab+c(a+b)$ với $a+b+c=n-1$.

Đặt $d= \frac {a+b} {2}$, ta có:

$P=a+b+ab+c(a+b) \leq 2d+d^2+(n-1-2d).(2d) = -3d^2+2nd = -3(d-k)^2+3k^2 \leq 3k^2$

Ta chỉ ra một cách xây dựng cấu hình có $3k^2$ đường đi.

Chia $n$ thành phố thành 3 nhóm $A,B,C$, mỗi nhóm gồm $k$ thành phố. Với mỗi bộ ba thành phố $X,Y,Z$ với $X \in A, Y \in B, Z \in C$, ta xây dựng đường một chiều theo quy tắc $X \Rightarrow Y \Rightarrow Z \Rightarrow X$. Dễ thấy cách xây này thoả mãn yêu cầu và có số đường đi là $3k^2$ (có $k^3$ bộ thành phố, ứng với mỗi bộ có $3$ đường một chiều, trong đó mỗi con đường được tính đúng $k$ lần).

Tóm lại, số con đường đi nhiều nhất có thể xây dựng với $n=3k$ thành phố là $3k^2$.




#481828 Trận 3 - Tổ hợp rời rạc

Đã gửi bởi maitienluat on 07-02-2014 - 23:32 trong Thi giải toán Marathon Chuyên toán 2014

Bài làm :

 

Gọi $X$ là thành phố có nhiều đường đi liên quan nhất ( theo nghĩa tổng số đường đi từ $X$ đến thành phố khác và từ thành phố khác đến $X$ là lớn nhất ).

Ta chia $209$ thành phố còn lại làm $3$ nhóm : Nhóm $A$ gồm tất cả các thành phố có đường đi một chiều tới $X$, nhóm $B$ gồm các thành phố mà từ $X$ có đường đi một chiều tới và nhóm $C$ bao gồm các thành phố còn lại (trừ X). Đặt $a,b,c$ lần lượt là số phần tử của $A, B, C$. Khi đó, số đường đi có liên quan tới $X$ là $a+b$ và $a+b+c =209$.

Ta có một số nhận xét sau

$\bullet$ Giữa các thành phố cùng thuộc nhóm A không có đường đi nối với nhau.

  C/minh: giả sử $A_1,A_2$ là 2 thành phố cùng thuộc $A$ sao cho có đường đi nối giữa $A_1$ và $A_2$, không mất tổng quát giả sử đó là đường đi từ $A_1$ tới $A_2$. Khi đó có đường đi từ $A_1$ tới $A_2$ và từ $A_2$ tới $X$, nên theo giả thiết không có đường đi từ $A_1$ tới $X$, vô lí vì $A_1 \in A $

  Tuơng tự, giữa các thành phố cùng thuộc nhóm B không có đường đi nối với nhau.

  Từ đó, số đường đi nối giữa các thành phố thuộc $A$ và $B$ không lớn hơn $ab$.

$\bullet$ Số đường đi liên quan tới các thành phố thuộc nhóm $C$ không lớn hơn $c(a+b)$.

   Thật vậy, vì nếu ngược lại thì theo nguyên lí Đirichlet tồn tại một thành phố thuộc $C$ có số đường đi liên quan lớn

hơn $a+b$, mâu thuẫn với cách chọn $X$.

Từ các nhận xét trên ta suy ra số con đường có thể xây dựng không lớn hơn: $a+b+ab+c(a+b)$.

Ta có $P = a+b+ab+c(a+b) \leq a+b+\frac {(a+b)^2} {4} + (209 -a-b)(a+b) = 2d + d^2+2d(209 -2d) = - 3(d-70)^2 +  3.70^2 \leq 14700$.

(với $d= \frac {a+b} {2} $)

Ta chỉ ra một cách xây dựng đường đi thoả mãn yêu cầu bài toán. 

Chia $210$ thành phố thành 3 nhóm $A,B,C$, mỗi nhóm có 70 thành phố. Với mỗi bộ ba thành phố $X,Y,Z$ với $X \in A, Y \in B, Z \in C$ thì ta xây dựng đường đi theo quy tắc $X \Rightarrow Y \Rightarrow Z \Rightarrow X$. Dễ thấy cách xây dựng này thoả yêu cầu bài. Khi đó số bộ thành phố là $70^3$, với mỗi bộ có $3$ con đường được xây và mỗi con đường được tính lặp lại $70$ lần nên số đường đi là $3.70^2=14700$.

Tóm lại, số đường đi nhiều nhất có thể là $14700$




#476731 Trận 1 - Số học

Đã gửi bởi maitienluat on 11-01-2014 - 21:26 trong Thi giải toán Marathon Chuyên toán 2014

Ta có 

$$ x^2 = y^2 + \sqrt{y+1} \Leftrightarrow x^2 - y^2 = \sqrt {y+1}$$

Do vế trái là số nguyên nên vế phải cũng là số nguyên $\Rightarrow y= k^2-1 , k \in \mathbb{N}, k \geq 1$

Thế lại vào phương trình ta được 

$$ x^2 = (k^2-1)^2 + k = k^4 - 2k^2 + k + 1$$

Do $k \geq 1$ nên $2k^2 \geq k + 1$

$$ \Rightarrow (k^2-1)^2 < x^2 = k^4 - 2k^2 + k + 1 \leq k^4$$

$$\Rightarrow 2k^2 = k+1 \Rightarrow k=1$$

Khi đó ta có $y = k^2 -1 = 0$ và $x^2=1 \Rightarrow x=1$

Thử lại thấy $(x;y) = (1;0)$ thoả đề.

Kết luận : $(x,y) = (1,0)$

 

$\Rightarrow 2k^2 = k+1 \Rightarrow k=1$

Chỗ này hãy viết rõ hơn như sau: $\Rightarrow x^2=k^4 \Rightarrow 2k^2 = k+1 \Rightarrow k=1$: trừ 0,5đ

$d=9,5$

$d_{mr}=0;d_{tl}=0;d_{t}=0$

$S=45,5$




#476670 $\sum \dfrac{(a+b)^2a_1b_1}{16S^2} \g...

Đã gửi bởi maitienluat on 11-01-2014 - 14:51 trong Hình học

Ta có  $\frac {AC'}{AB}=\frac{AC}{AC+BC} \Rightarrow AC' = \frac {bc}{a+b}$

Tương tự $AB'=\frac {bc}{c+a}$

Áp dụng định lý cosine cho tam giác $AB'C'$ ta được:

$$a_1^2=AC'^2+AB'^2-2AC'.AB'.\cos A \geq 2AC'AB'(1-\cos A)=\frac {2b^2c^2}{(a+c)(a+b)}.2\sin^2 {\frac {A}{2}}=\frac {4bc(p-b)(p-c)} {(a+c)(a+b)}$$

Tương tự ta thu được các đánh giá với $b_1^2$ và $c_1^2$.

Bây giờ, áp dụng BĐT AM-GM cho vế trái của BĐT đã cho, ta sẽ chứng minh:

$$3. \sqrt[3] {(a+b)^2(b+c)^2(c+a)^2.a_1^2.b_1^2c_1^2} \geq 16S^2$$

Sử dụng các đánh giá ở trên, ta đưa về chứng minh

$$3. \sqrt[3] {64a^2b^2c^2(p-a)^2(p-b)^2(p-c)^2}\geq 16p(p-a)(p-b)(p-c)$$

$$\Leftrightarrow 27a^2b^2c^2 \geq 64 p^3(p-a)(p-b)(p-c)$$ (*)

Đặt $x=p-a,y=p-b,z=p-c$ thì $x,y,z > 0$ và

$$(*) \Leftrightarrow 27(x+y)^2(y+z)^2(z+x)^2 \geq 64xyz(x+y+z)^3$$

Hiển nhiên đúng theo 2 BĐT quen thuộc sau

$$9(x+y)(y+z)(z+x) \geq 8(x+y+z)(xy+yz+zx)$$

$$(xy+yz+zx)^2 \geq 3xyz(x+y+z)$$

Kết thúc chứng minh. Đẳng thức xảy ra khi và chỉ khi $ABC$ là tam giác đều.




#474959 [VMO 2014] Ngày 1 - Bài 1 - DÃY SỐ

Đã gửi bởi maitienluat on 03-01-2014 - 12:40 trong Dãy số - Giới hạn

 

ĐỀ THI CHỌN HỌC SINH GIỎI CẤP QUỐC GIA
MÔN TOÁN NĂM 2014
Thời gian làm bài: 180 phút.
Ngày thi thứ nhất (03/01/2014)

Bài 1. (5 điểm) Cho 2 dãy số thực dương $(x_{n}),(y_{n})$ xác định bởi $x_{1}=1,y_{1}=\sqrt{3}$

$$\left\{\begin{matrix} x_{n+1}y_{n+1}-x_{n}=0\\x_{n+1}^2+y_{n} =2 \end{matrix}\right.\forall n=1,2,3$$

Chứng minh rằng hai dãy số trên hội tụ và tìm giới hạn của chúng.

Từ giả thiết ta suy ra :

$$\left\{\begin{matrix} x_1=2\sin {\frac {\pi}{6}}, &y_1=2\cos {\frac {\pi}{6}} \\ x_{n+1}=\sqrt{2-y_n} & \\ y_{n+1}=\frac {x_n}{x_{n+1}} & \end{matrix}\right.$$

Từ đó, bằng quy nạp ta chứng minh được $$\left\{\begin{matrix} x_n=2\sin {\frac {\pi}{3.2^n}}\\ y_n=2\cos {\frac {\pi}{3.2^n}} \end{matrix}\right.$$

Do $\cos x$ nghịch biến trên $(0; \pi)$ nên ta suy ra $(y_n)$ tăng. Mặt khác, $(y_n)$ bị chặn trên bởi $2$ nên nó hội tụ.

Tương tự, do $\sin x$ đồng biến trên $(0; \pi)$ nên $(x_n)$ giảm và bị chặn dưới bới $0$ nên cũng hội tụ.

Do tính liên tục của $\cos x$ và $\sin x$ nên ta suy ra

$$\lim x_n = \lim 2\sin {\frac {\pi}{3.2^n}}=2\sin \lim {\frac {\pi}{3.2^n}}= 2\sin 0 = 0$$

$$\lim y_n = \lim 2\cos {\frac {\pi}{3.2^n}}=2\cos \lim {\frac {\pi}{3.2^n}}= 2\cos 0 = 2$$

 


 




#467198 ĐỀ KIỂM TRA TRƯỜNG ĐÔNG TOÁN HỌC MIỀN BẮC

Đã gửi bởi maitienluat on 27-11-2013 - 21:32 trong Thi HSG cấp Tỉnh, Thành phố. Olympic 30-4. Đề thi và kiểm tra đội tuyển các cấp.



Bài 1. 

Cho dãy số $(a_n)_{n\geq 1}$ xác định bởi $a_1=\frac{3}{2}$ và :

$$a_{n+1}=a_n-\frac{3n+2}{2n(n+1)(2n+1)}\,\,\, \forall n\geq 1$$

Tìm $\text{lim}_{n\to \infty} a_n$.

 

Từ công thức truy hồi ta có

$$a_{n+1}=a_n+ \frac {1} {2n+1} + \frac {1} {2n+2} - \frac {1} {n}$$

Suy ra $$a_{n+1}=a_1+\sum_{k=1}^{n}\left ( \frac {1}{2k+1}+\frac{1}{2k+2} -\frac {1}{k}\right )=\frac {3}{2}+\sum_{k=3}^{2n+2}\frac {1}{k}-\sum_{k=1}^n \frac{1}{k}=\sum_{k=0}^{n+1} \frac{1}{n+1+k}$$

$$\Rightarrow a_n=\sum_{k=0}^{n}\frac{1}{n+k}$$

Xét hàm số $f(x)=\frac {1} {1+x}$. Tổng tích phân cấp $n$ của hàm số $f$ trên $[0,1]$ là

$$S_n = \frac {1} {n} \sum_{i=1}^n f(\frac {1} {n}) = a_n - \frac {1} {n}$$

Theo định lí cơ bản của tích phân $$\lim S_n=\int_{0}^{1}\frac {1}{x+1}dx=\ln2$$

Mà $\lim \frac {1} {n} = 0$ nên ta suy ra $$\lim a_n = \lim S_n = \ln 2$$.




#464137 Cm $\lim\limits_{n\to +\infty} \frac...

Đã gửi bởi maitienluat on 13-11-2013 - 20:03 trong Dãy số - Giới hạn

Cho dãy số $\{x_n\}$ thỏa $x_1=1;x_{n+1}=\frac{n}{x_n}+\frac{x_n}{n}$. Chứng minh $$\lim\limits_{n\to +\infty} \frac{x_n^2}{n}=1; \; \lim\limits_{n\to +\infty}(x_n^2-n)=\frac{1}{2}$$

$\bullet$ Chứng minh $\lim_{n\rightarrow +\infty }\frac{x_n^2}{n}=1$

Ta chứng minh bằng quy nạp rằng $\sqrt{n}<x_n<\sqrt{n}+1$.

Với $n=1,n=2$ đúng. Giả sử nó đúng tới $n=k \geq 2 \Rightarrow \sqrt{k}<x_k< \sqrt{k}+1$.

Xét hàm số $f_k(x)= \frac {x} {k} + \frac {k} {x}$ trên $(\sqrt{k};\sqrt{k}+1)$

Ta có $f'(x)= \frac {x^2-k^2} {x^2} < 0 $ do $x< \sqrt{k}+1<k (k \geq 2)$

Suy ra $f(\sqrt{k})> f(x_k) > f(\sqrt{k}+1)$

Mặt khác dễ dàng kiểm tra được rằng $f(\sqrt{k}) < \sqrt{k+1} +1$ và $f(\sqrt{k}+1) > \sqrt{k+1}$

nên ta suy ra $\sqrt{k+1}+1 > x_{k+1} > \sqrt{k+1}$.

Từ đó áp dụng nguyên lý kẹp ta thu được $\lim_{n\rightarrow +\infty}\frac {x_n}{\sqrt{n}}=1$.

Vậy ta có đpcm.

$\bullet$ Chứng minh $\lim_{n\rightarrow +\infty} (x_n^2-n) = \frac {1} {2}$.

Lập dãy $(y_n) y_n=x_n^2 \Rightarrow \left\{\begin{matrix} y_1=1 & \\ y_{n+1}=\frac {y_n}{n^2}+\frac{n^2}{y_n}+2 & \end{matrix}\right$

Bằng cách tương tự như câu a, ta chứng minh được BĐT sau với mọi $n \geq 2$

$$n+\frac{1}{2}<y_n<n+\frac{1}{2}+\frac{5n^2+4+1}{4n^3-2n^2-8n-4}$$

Áp dụng nguyên lí kẹp ta có đpcm.




#460922 Chứng minh rằng: $\sum\frac{1}{\sqrt{m_a}}\ge\s...

Đã gửi bởi maitienluat on 30-10-2013 - 20:04 trong Các bài toán Lượng giác khác

 Bổ đề: $m_a+m_b+m_c \leq  \frac {9R} {2}$

C/minh:

  Sử dụng kết quả quen thuộc $\sin ^2 A+ \sin ^2 B+ \sin ^2 C  \leq \frac {9} {4}$, ta có

$$(m_a+m_b+m_c)^2 \leq 3(m_a^2+m_b^2+m_c^2) = \frac {9} {4} (a^2+b^2+c^2) = 9R^2(\sin ^2 A+ \sin ^2 B + \sin ^2 C ) \leq \frac {81} {4} R^2$$

 $\Rightarrow$ đpcm.

Trở lại bài toán. Áp dụng BĐT Cauchy-Schwarz và bổ đề trên, ta được

$$\sum \frac {1} {\sqrt{m_a}} \geq \frac {9} {\sum \sqrt {m_a}} \geq \frac {9} {\sqrt {3(\sum m_a)}} \geq \frac {9} {\sqrt {3. \frac {9R} {2}}} = \sqrt {\frac {6} {R}}$$

Đẳng thức xảy ra khi và chỉ khi tam giác $ABC$ đều.




#449471 Topic về số học, các bài toán về số học.

Đã gửi bởi maitienluat on 11-09-2013 - 21:14 trong Số học

Bài 50: Tìm tất cả các số nguyên dương $m$ và $n$ sao cho

$\left \lfloor \frac{m^2}{n} \right \rfloor+\left \lfloor \frac{n^2}{m} \right \rfloor=\left \lfloor \frac{m}{n}+\frac{n}{m} \right \rfloor+mn$

Ta có nhận xét sau: Cho $n$ là một số tự nhiên và $a$ là một số thực. Khi đó nếu $a \geq n$ thì $\left \lfloor a \right \rfloor\geq n$ và nếu $a \leq n$ thì $\left \lfloor a \right \rfloor\leq n$

Trở lại bài toán. KMTQ giả sử $m \geq n$

$\bullet$ TH1: $n^2 \geq m$

Khi đó ta có $(m^2-n)(n^2-m)\geq 0 \Rightarrow mn+1 \geq \frac{m^2}{n}+\frac{n^2}{m}$

Suy ra $\left \lfloor \frac{m^2}{n} \right \rfloor+\left \lfloor \frac {n^2}{m} \right \rfloor=mn+\left \lfloor \frac{m}{n}+\frac{n}{m} \right \rfloor \geq mn+2 \geq \frac {m^2}{n}+\frac{n^2}{m}+1>\left \lfloor \frac {m^2}{n}+\frac {n^2}{m} \right \rfloor \geq \left \lfloor \frac{m^2}{n} \right \rfloor+\left \lfloor \frac {n^2}{m} \right \rfloor$.

Vây trong trường hợp này pt vô nghiệm

$\bullet$ TH2: $n^2 < m$

Đặt $m=an^2+b, b^2=cn+d$. Bỏ qua truờng hợp đơn giản khi $b=0$.

Khi đó ta dễ dàng có được :

$\left \lfloor \frac{m^2}{n} \right \rfloor=a^2n^3+2anb+c; \left \lfloor \frac{n^2}{m} \right \rfloor=0$

Mặt khác để ý rằng $b^2<(c+1)n \Rightarrow b<\sqrt{(c+1)n}\leq \frac {n+c+1}{2} \Rightarrow b \leq \frac {n+c}{2}$

ta dễ dàng chứng minh được $\frac{m}{n}+\frac{n}{m}=n+\frac{(2a-1)bn^2+b^2+n^2}{an^3+bn}< n+c+1 \Rightarrow \left \lfloor \frac{m}{n}+\frac{n}{m} \right \rfloor \leq n+c$

Suy ra $a^2n^3+2abn+c\leq n+c+(an^2+b)n \Leftrightarrow (a^2-a)n^3+(2ab-b-1)n \leq 0$

Mà do $a,b \geq 1$ nên $a^2-a \geq 0$ và $2ab-b-1 \geq 0$ nên ta suy ra $\left\{\begin{matrix} a^2-a=0 & \\ 2ab-b-1=0 & \end{matrix}\right.\Rightarrow \left\{\begin{matrix} a=1 & \\ b=1 & \end{matrix}\right.$. Thử lại thấy $(m,n)=(k^2+1,k)$ thỏa.

KL: $(m,n)=(k^2+1,k);(k,k^2+1)$ với $k \in \mathbb{N}$.




#449451 $x_{n}=\frac{1}{n}\sqrt[n]{...

Đã gửi bởi maitienluat on 11-09-2013 - 20:23 trong Dãy số - Giới hạn



1) $lim \left ( \frac{1}{n\sqrt{1+n^{2}}}+\frac{2}{n\sqrt{2+n^{2}}}+...+\frac{n}{n\sqrt{n+n^{2}}} \right )$

 

Đặt $S_n = \left ( \frac{1}{n\sqrt{1+n^{2}}}+\frac{2}{n\sqrt{2+n^{2}}}+...+\frac{n}{n\sqrt{n+n^{2}}} \right )$

Ta có $\frac{k}{n} \leq \frac{k}{\sqrt {n^2+k}} \leq \frac{k}{\sqrt {n^2+n}}$

$\Rightarrow \frac{1}{n}.\sum \frac{k}{n}\leq S_n \leq \frac{1}{n}.\sum \frac{k}{\sqrt {n^2+n}}$

$\Rightarrow \frac{n+1}{2n}\leq S_n\leq \frac{n+1}{2\sqrt {n^2+n}}$

Mà $\lim _{n\rightarrow +\infty }\frac{n+1}{2n}=\lim _{n\rightarrow +\infty }\frac{n+1}{2\sqrt {n^2+n}}=\frac{1}{2}$

nên theo nguyên lý kẹp ta có $\lim _{n\rightarrow +\infty }S_n = \frac{1}{2}$

 

 



2) $x_{n}=\frac{1}{n}\sqrt[n]{(n+1)(n+2)...(n+n)}$

Từ hệ thức ta suy ra $x_{n+1}^{n+1}=2x_n^n .\frac{n^n}{(n+1)^{n+1}}.(2n+1)$ (*)

$\Rightarrow \left ( 1+ \frac{1}{n} \right )^n.x_{n+1}^{n+1} =x_n^{n+1}.\frac {2n}{n+1}.\sqrt[n]{(n+1)(n+2)...(2n)}$

Để ý rằng $\left ( 1+ \frac{1}{n} \right )^n< 3 \forall n$

và $\frac {2n}{n+1}.\sqrt[n]{(n+1)(n+2)...(2n)}> \frac{2n}{n+1}.n> 3 \forall n\geq 3$

Nên kể từ số hạng thứ 3 trở đi thì $(x_n)$ là dãy giảm và bị chặn dưới bởi $0$ nên tồn tại giới hạn hữu hạn $L \geq 0$.

Chuyển (*) qua giới hạn ta được

$L = \lim_{n\rightarrow +\infty}\left ( 2(2n+1).\frac{n^n}{(n+1)^{n+1}} \right )=\lim_{n\rightarrow +\infty} \left ( 2. \frac {2n+1}{n+1}. \frac {1}{(1+\frac {1}{n})^n}\right )= \frac {4}{e}$

=====================================

Ngoài ra ta cũng có thể dùng tổng tích phân như sau:

$x_n = \sqrt[n]{\prod _{k=1}^n\left ( 1+\frac{k}{n} \right )}\Rightarrow y_n= \ln x_n = \frac{1}{n}.\sum_{k=1}^n \ln {\left (1+\frac{k}{n} \right )}$

Xét hàm số $f(x)=\ln {(1+x)}$ trên $[0,1]$. Chia đoạn $[0,1]$ thành $n$ đoạn bằng nhau bởi các điểm chia $a_k=\frac {k} {n}, 1 \leq k \leq n$.

Khi đó tổng tích phân cấp $n$ trên đoạn $[0,1]$ của $f(x)$ là:

$S_n=\sum_{k=1}^nf(a_k)(a_k-a_{k-1})=\frac {1}{n}.\sum_{k=1}^n \ln {\left ( 1 + \frac {k}{n} \right )}=y_n$

Theo định lý cơ bản của tích phân $\lim_{n \rightarrow +\infty}S_n = \int_{0}^{1}\ln(1+x)dx$

Thực hiện phép đổi biến $x=u^2$ và lấy tích phân từng phần ta tính được $\int_{0}^{1}\ln{(1+x)}dx=2\ln2 -1$

Suy ra $\lim_{n \rightarrow +\infty}S_n=\lim_{n \rightarrow +\infty}y_n= 2\ln2 -1$

Do hàm số $\ln {x}$ liên tục trên $(0,+\infty )$ nên ta có $\lim_{n \rightarrow +\infty}x_n=e^{\lim_{n \rightarrow +\infty}y_n}=e^{2\ln2-1}=\frac {4}{e}$




#444146 Topic nhận đề về Bất đẳng thức và cực trị

Đã gửi bởi maitienluat on 19-08-2013 - 21:28 trong Bài thi đang diễn ra

1. Họ và tên thật: Mai Tiến Luật

2. Đang học lớp: 11T, trường: THPT chuyên Lê Quý Đôn, thành phố Quy Nhơn, tỉnh Bình Định

3. Đề:

Cho số tự nhiên $n \geq 2$. Chứng minh rằng với mọi số thực dương $a_1, a_2, ..., a_n, x_1, x_2, ..., x_n$ thoả mãn $\sum_{1 \geq i \geq j \geq n} x_i.x_j= \frac {n(n-1)} {2}$ , ta có :

$$\frac {a_1} {a_2+a_3+...+a_n} . (x_2+x_3+...+x_n) + ... + \frac {a_n} {a_1+a_2+...+a_{n-1}}.(x_1+x_2+...+x_{n-1}) \geq n$$

4. Đáp án

Đặt $S=a_1+a_2+...+a_n$. Sử dụng điều kiện ta viết lại BĐT dưới dạng:

 $$\sum_{i=1}^{n} \frac {a_i} {S-a_i} . (x_1+...+x_{i-1}+x_{i+1}+x_n) \geq \sqrt {\frac {2n} {n-1} . \sum_{1 \geq i \geq j \geq n} x_i.x_j} $$

Ta có thể chuẩn hoá cho $\sum_{i=1}^{n} x_i =1$. BĐT trở thành:

$$\sum_{i=1}^{n} \frac {a_i} {S-a_i} . (1-x_i) \geq \sqrt {\frac {2n} {n-1} . \sum_{1 \geq i \geq j \geq n} x_i.x_j} $$

$$\Leftrightarrow 2\sqrt {\frac {n} {2(n-1)} . \sum_{1 \geq i \geq j \geq n} x_i.x_j}+ \sum_{i=1}^{n} \frac {a_i} {S-a_i} . x_i \leq \sum_{i=1}^{n} \frac {a_i} {S-a_i} $$ (1)

Áp dụng BĐT Cauchy-Schwarz ta có

$${VT^2(1) \leq (\sum_{i=1}^n x_i^2 +2 \sum_{1 \geq i \geq j \geq n} x_i.x_j})\left ( \frac{n}{2(n-1)}+\frac{n}{2(n-1)}+\sum_{i=1}^n\frac{a_i^2}{(S-a_i)^2} \right )= \frac {n} {n-1} + \sum_{i=1}^n\frac{a_i^2}{(S-a_i)^2}$$

Để chứng minh (1) ta chỉ cần chỉ ra

$$\frac {n} {n-1} + \sum_{i=1}^n\frac{a_i^2}{(S-a_i)^2} \leq \left (\sum_{i=1}^n \frac{a_i}{S-a_i} \right )^2$$

$$\Leftrightarrow \sum_{1\leq i<j \leq n} \frac{a_ia_j}{(S-a_i)(S-a_j)}\geq \frac{n}{2(n-1)}$$ (2)

Áp dụng BĐT Cauchy-Schwarz ta có

$$VT(2) = \sum_{1\leq i<j \leq n} \frac{a_i^2a_j^2}{a_ia_j(S-a_i)(S-a_j)} \geq \frac{(\sum a_ia_j)^2}{\sum a_ia_j(S-a_i)(S-a_j)}$$

Đặt $b_i=a_i(S-a_i)$. Theo BĐT AM-GM ta dễ dàng có được $$(n-1)\sum b_i^2\geq 2\sum b_ib_j$$

$$\Rightarrow (\sum b_i)^2 \geq (2+\frac{2}{n-1})(\sum b_ib_j)=\frac{2n}{n-1}(\sum b_ib_j)$$

Từ đó, để ý rằng $2\sum a_ia_j= \sum a_i(S-a_i)$ ta suy ra

$$VT(2) \geq \frac{\left ( \sum b_i \right )^2}{4\sum b_ib_j}\geq \frac{1}{4}.\frac{2n}{n-1}=\frac{n}{2(n-1)}= VP(2)$$

Vậy BĐT (2) được chứng minh, tức là (1) được chứng minh. Đẳng thức xảy ra khi và chỉ khi $a_1=a_2=...=a_n > 0$ và $x_1=x_2=...=x_n=1$.

 

 

 

 

 

 

 




#430871 $$\sum_{cyc}\frac{\sin^2 A}...

Đã gửi bởi maitienluat on 26-06-2013 - 21:54 trong Bất đẳng thức - Cực trị

Bài toán 1: Với tam giác $ABC$ bất kỳ,ta có $\sum_{cyc}\frac{\sin^2 A}{\cos B\cos C}\ge\left(\frac{6r}{R}\right)^2$

 Có lẽ điều kiện nên là tam giác $ABC$ nhọn, vì chẳng hạn với $A=\frac{2\pi}{3},B=C=\frac{\pi}{6}$ thì vế trái âm và nếu tam giác $ABC$ vuông thì vế trái k xác định.

=============

Ta giải bài toán khi $ABC$ là tam giác nhọn. BĐT đã cho tương đương với:

$$\sum \frac{R^{2}\sin^{2} {A}}{\cos B \cos C}\geq 36r^{2}$$

Ta có $$\frac{R^{2}\sin^{2} {A}}{\cos B \cos C}= \frac{\frac{a^{2}}{4}}{\frac{(b^2+a^2-c^2)(c^2+a^2-b^2)}{4a^2bc}}=\frac{a^4bc}{a^4-(b^2-c^2)^2}\geq bc$$

và $$36r^2=36\frac {S^2}{p^2}=36\frac {(p-a)(p-b)(p-c)}{p}=9\frac {(a+b-c)(b+c-a)(c+a-b)} {a+b+c}$$

nên ta viết lại BĐT thành

$$(a+b+c)(ab+bc+ca)\geq 9(a+b-c)(b+c-a)(c+a-b)$$

Mặt khác theo AM-GM dễ có

$$(a+b+c)(ab+bc+ca)\geq 9abc$$

$$abc\geq (b+c-a)(c+a-b)(a+b-c)$$

Ta có đpcm. Đẳng thức xảy ra $\Leftrightarrow a=b=c \Leftrightarrow ABC$ đều.




#430627 Dãy số-Giới hạn Tuyển tập sưu tầm các bài toán từ Mathlinks.ro

Đã gửi bởi maitienluat on 25-06-2013 - 22:29 trong Dãy số - Giới hạn

Bài toán 34: Cho phương trình $ x^n=x^{n-1}+...+x+1 $.

  1. Chứng minh rằng PT trên có duy nhất nghiệm thực dương,gọi nghiệm đó là $x_n$.
  2.  
  3. Chứng minh $\lim_{n \to +\infty}x_n=2$
  4.  
  5. Tính $\lim_{n \to +\infty}n(2-x_n)$.

1. Đặt $f_{n}(x)=x^{n}-x^{n-1}-...-1$

Ta chứng minh bằng quy nạp rằng$f_{n}(x)$ có một nghiệm dương duy nhất $x_{n}$ thoả $1 \leq x_{n} \leq 2$

Với $n=1,n=2$ khẳng định đúng. Giả sử nó đúng tới $n$.

Để ý rằng $f_{n+1}(x)=f_{n}(x)+x^{n+1}-2x^{n}$ ta có

  $f_{n+1}(x_{n})=f_{n}(x_{n})+x_{n}^{n}(x_{n}-2)<0$

  $f_{2}=1>0$

Suy ra $f_{n+1}(x)$ có một nghiệm nằm giữa $x_{n}$ và $2$. Tính dương và duy nhất của nó là hiển nhiên (theo quy tắc Descartes về số nghiệm dương của một phương trình đa thức).

Mặt khác $x_{n+1} > x_{n} >1$. Tóm lại mệnh đề quy nạp đúng với mọi n.

 

2. Do $f_{n}(2)=1$ và $f_{n}(x_{n})=0$ nên theo định lý Lagrange ta có

    $$1=\left | f_{n}(x_{n})-f_{n}(2) \right |=\left | f_{n}^{'}(a) \right |.\left | x_{n}-2 \right |$$

với $a \in (x_{n},2) \Rightarrow a>x_{n-1}>1$ Từ đó $f_{n-1}(a)>0$ nên 

$$f_{n}^{'}(a)=na^{n-1}-(n-1)a^{n-2}-...-1>n(a^{n-2}+...+1)-(n-1)a^{n-2}-...-1=a^{n-2}+2a^{n-3}+...+(n-1)>1+2+...+(n-1)=\frac {n(n-1)}{2}$$

Suy ra $$0<\left | x_{n}-2 \right |=\frac{1}{\left | f_{n}^{'}(a) \right |}< \frac{2}{n(n-1)}\rightarrow 0$$

Theo nguyên lý kẹp ta có $$\lim_{n\rightarrow +\infty }x_{n}=2$$

 

3. Cũng từ đánh giá ở trên ta có

$$0<n(2-x_{n})=\frac{n}{f_{n}^{'}(a)}< \frac{2}{n-1}\rightarrow 0$$

nên theo nguyên lý kẹp $$\lim_{n\rightarrow +\infty}n(2-x_{n})=0$$




#430137 Số học -Tuyển tập các bài toán sưu tầm từ Mathlinks.ro

Đã gửi bởi maitienluat on 24-06-2013 - 08:16 trong Số học

Ta dễ dàng có nghiệm của phương trình là $(a;b)=(u_{n};u_{n+1})$ với dãy $\{u_n\}$ được xác định bởi $u_{0}=1;u_{1}=4$, $u_{n+1}=6.u_n-u_{n-1}$

$\bullet$ với $n=4$ ta có ngay nghiệm khởi đầu bằng $(2;2)$. 

Phương trình trở thành $a^2+b^2-3ab=-4$.

Làm tương tự trên là có ngay nghiệm của phương trình là $(a;b)=(u_{n};u_{n+1})$ với dãy $\{u_n\}$ được xác định bởi $u_{0}=2;u_{1}=2$, $u_{n+1}=3.u_n-u_{n-1}$....

Spoiler

Hình như cậu làm sót nghiệm rồi Đạt à. Với trường hợp $n=7$ ta còn có 1 bộ nghiệm khởi đầu là ($1;2$) và do đó ta còn một dãy nghiệm xác định bới $(a,b)=(u_{n};u_{n+1})$ với $u_{1}=1;u_{2}=2;u_{n+1}=6u_{n}-u_{n-1}$.

 

======================

 

Có ai có tài liệu về pt Markov ở trên k?

=======================




#428572 Chứng minh rằng $A1B1,A2B2,A3B3$ đồng quy.

Đã gửi bởi maitienluat on 18-06-2013 - 15:23 trong Hình học

a) Ta có $\frac{B_{3}O_{1}}{B_{2}O_{1}}.\frac{B_{2}O_{1}}{B_{2}O_{3}}.\frac{B_{1}O_{3}}{B_{1}O_{2}}=1$

Nên theo định lý Ceva đảo, $O_{1}B_{1}, O_{2}B_{2}, O_{3}B_{3}$ đồng quy tại một điểm (là $I$).

b) Dễ kiểm tra rằng điểm $I$ trên là tâm tỉ cự của hệ {$O_{1},O_{2},O_{3}$} hệ số $\left ( \frac{1}{R_{1}},\frac{1}{R_{2}},\frac{1}{R_{3}} \right )$ với .

$\Rightarrow \left ( \frac{1}{R_{1}}+\frac{1}{R_{2}}+\frac{1}{R_{3}} \right )\overrightarrow{KI}=\frac{1}{R_{1}}\overrightarrow {KO_{1}}+\frac{1}{R_{2}}\overrightarrow {KO_{2}}+\frac{1}{R_{3}}\overrightarrow {KO_{3}}$

Mặt khác theo cách chọn điểm $K$ thì

$\frac{1}{R}\overrightarrow {KO}=\frac{1}{R_{1}}\overrightarrow {KO_{1}}+\frac{1}{R_{2}}\overrightarrow {KO_{2}}+\frac{1}{R_{3}}\overrightarrow {KO_{3}}$

Suy ra $\overrightarrow {KO}=R\left ( \frac{1}{R_{1}}+\frac{1}{R_{2}}+\frac{1}{R_{3}} \right )\overrightarrow {KI}$

Tức là $K,I,O$ thẳng hàng.




#428452 Chứng minh rằng $A1B1,A2B2,A3B3$ đồng quy.

Đã gửi bởi maitienluat on 18-06-2013 - 08:45 trong Hình học

Gọi $O,O_{1},O_{2},O_{3},R,R_{1},R_{2},R_{3}$ lần lượt là tâm và bán kính của các đường tròn $m,m_{1},m_{2},m_{3}$ và $K$ là tâm tỉ cự của hệ điểm {$O,O_{1},O_{2},O_{3}$} hệ số {$\frac{-1} {R}, \frac{1} {R_{1}}, \frac{1} {R_{2}}, \frac{1} {R_{3}}$}.

Từ giả thiết suy ra $A_{1},O_{1},O$ thẳng hàng theo thứ tự đó nên

$-A_{1}O_{1}\vec {A_{1}O}+A_{1}O\vec {A_{1}O_{1}}=\vec {0}$

$\Rightarrow -R_{1}.\vec {A_{1}O}+R\vec {A_{1}O_{1}}= \vec {0}$

$\Rightarrow \frac {-1} {R} \vec {A_{1}O}+ \frac {1} {R_{1}} \vec {A_{1}O_{1}}= \vec {0}$ (1)

Tương tự từ $B_{1},O_{3},O_{2}$ thẳng hàng ta có $\frac {1} {R_{2}} \vec {B_{1}O_{2}} + \frac {1} {R_{3}} \vec {B_{1}O_{3}}= \vec {0}$ (2)

Từ (1), (2) và để ý rằng:

$\frac{-1}{R}\overrightarrow{KO}+\frac{1}{R_{1}}\overrightarrow{KO_{1}}+\frac{1}{R_{2}}\overrightarrow{KO_{2}}+\frac{1}{R_{3}}\overrightarrow{KO_{3}}=\overrightarrow{0}$

ta có $\frac{-1}{R}\overrightarrow{KA_{1}}+\frac{1}{R_{1}}\overrightarrow{KA_{1}}+\frac{1}{R_{2}}\overrightarrow{KB_{1}}+\frac{1}{R_{3}}\overrightarrow{KB_{1}}=\overrightarrow{0}$

Suy ra $K \in A_{1}B_{1}$. Tương tự $K \in A_{2}B_{2}$,$K \in A_{3}B_{3}$.

Vậy $A_{1}B_{1}, A_{2}B_{2}, A_{3}B_{3}$ đồng quy.




#428327 Chứng minh rằng tồn tại ba đỉnh của nó tạo thành 1 tam giác có diện tích khôn...

Đã gửi bởi maitienluat on 17-06-2013 - 21:07 trong Hình học

Trước tiên ta có 2 bổ đề sau:

 $\bullet$ Cho $\alpha _{i} \in (0,\pi )$. Khi đó ta có $\prod_{i=1}^{n}\sin \alpha_{i}\leq \left (\sin \frac{\sum_{i=1}^{n}\alpha_{i}}{n} \right)^{n}$

 $\bullet$ Với $\alpha _{i} \in (0,\pi )$ thì $\alpha \geq \sin \alpha $

Trở lại bài toán.

Gọi $a_{i}, b_{i}, c_{i}$ lần lượt là độ dài các cạnh, góc xen giữa hai cạnh $a_{i}$ và $a_{i+1}$, diện tích tam giác tạo thành bởi hai cạnh $a_{i}$ và $a_{i+1}$.

Khi đó ta có $\prod_{i=1}^{n}c_{i} = \prod_{i=1}^{n} \left ( \frac{1} {2} a_{i}a_{i+1} \sin b_{i} \right ) = \frac{1} {2^{n}} \prod_{i=1}^{n}a_{i}^{2} \sin b_{i}$.

Theo BĐT AM-GM ta có $$\prod_{i=1}^{n}a_{i} \leq  \left ( \frac {\sum_{i=1}^{n}a_{i}} {n} \right )^{n} = \left ( \frac{4} {n} \right)^{n}$$

Theo 2 bổ đề và đề ý rằng tổng các góc trong của một n-giác bằng $(n-2) \pi$ ta có $$\prod_{i=1}^{n} \sin b_{i} \leq \left (\sin \frac{\sum_{i=1}^{n}b_{i}}{n} \right)^{n} = \left ( \sin \frac {(n-2)\pi} {n} \right)^{n} = \left ( \sin \frac {2\pi} {n} \right)^{n} \leq  \left ( \frac {2\pi} {n} \right)^{n}$$.

Từ đó $$\prod_{i=1}^{n}c_{i} \leq \frac {1} {2^{n}}  \left ( \frac{4} {n} \right)^{2n} \left ( \frac {2\pi} {n} \right)^{n}$$

Suy ra tồn tại $i$ sao cho $c_{i} \leq \frac {1} {2}  \left ( \frac{4} {n} \right)^{2} \left ( \frac {2\pi} {n} \right) = \frac {16\pi} {n^{3}}.$

Bài toán được chứng minh.




#426917 Dãy số-Giới hạn Tuyển tập sưu tầm các bài toán từ Mathlinks.ro

Đã gửi bởi maitienluat on 13-06-2013 - 21:35 trong Dãy số - Giới hạn

====================

Bài toán 29: Cho dãy số $\{a_n \}$ thỏa $ a_1 = 1, a_{2n}= a_n+1, a_{2n+1}=\frac{1}{a_{2n}} $.Chứng minh rằng tất cả các số hữu tỷ đều là số hạng của dãy này.

-68-

Theo em đề nên là các số hữu tỷ dương vì {$a_{n}$} là dãy dương.

 

=============================

Ta chứng minh bằng quy nạp

Gọi $S(m)$ là mệnh đề $\forall p=\frac{r}{s}, gcd(r,s)=1, r < m, s < m$ , $\exists n$ sao cho $a_{n}=p$.

Với $m=1$ thì $p=1=a_{1}$, mệnh đề $S(1)$ đúng.

Giả sử mệnh đề đúng tới $m-1$. Ta chứng minh $S(m)$ đúng. Chỉ cần xét $p\neq 1$.

 $\bullet$ TH1: $p= \frac {m} {s}$ với $s < m$

  Ta có $m-s < m, s < m$ nên theo giả thiết quy nạp tồn tại $n$ sao cho $p-1= \frac {m-s} {s} = a_{n} \Rightarrow p=a_{n}+1=a_{2n} $.

 $\bullet$ TH2: $p= \frac {s} {m}$ với $s < m$

  Ta có $\frac {1-p} {p} = \frac {m-s} {s}$ nên theo giả thiết quy nạp tồn tại $n$ sao cho $\frac {1-p} {p} = a_{n} \Rightarrow \frac {1} {p} =a_{n}+1=a_{2n} \Rightarrow p=a_{2n+1}$.

Mệnh đề được chứng minh. Từ đây dễ dàng suy ra kết quả bài toán $\square$




#425026 f(a) = b, f(b) = c, f(c) = a

Đã gửi bởi maitienluat on 08-06-2013 - 11:26 trong Đa thức

Giả sử $f(x)$ là đa thức hệ số nguyên thoả $f(a)=b,f(b)=c,f(c)=a$ với $a,b,c$ là 3 số nguyên phân biệt.

Do $f(x)$ là đa thức hệ số nguyên và $a,b,c$ là các số nguyên phân biệt nên ta có

$b-c=f(a)-f(b) \vdots a-b \Rightarrow \left | b-c \right | \geq \left | a-b \right |$

Tương tự ta có $ \left | c-a \right | \geq \left | b-c \right |$, $\left | a-b \right | \geq \left | c-a \right |$

Từ đây suy ra $\left | b-c \right |=\left | c-a \right |=\left | a-b \right |$

Mà $a\neq b$ nên từ $\left | b-c \right |=\left | c-a \right |$ ta suy ra $b-c=c-a \Leftrightarrow a=2c-b$ (1)

Tương tự từ $\left | c-a \right |=\left | a-b \right |$ ta suy ra $c-a=a-b \Leftrightarrow c=2a-b$ (2)

Từ (1) và (2) ta có $a=c$, mâu thuẫn

Vậy k tồn tại đa thức hệ số nguyên thoả đề.